K
Khách

Hãy nhập câu hỏi của bạn vào đây, nếu là tài khoản VIP, bạn sẽ được ưu tiên trả lời.

4 tháng 9 2021

Ủa bị lỗi hả:v? undefined

AH
Akai Haruma
Giáo viên
13 tháng 4 2021

Lời giải:

Bạn nhớ tới bổ đề sau: Với $a,b>0$ thì $a^3+b^3\geq ab(a+b)$.

Áp dụng vào bài:

$5a^3-b^3\leq 5a^3-[ab(a+b)-a^3]=6a^3-ab(a+b)$

$\Rightarrow \frac{5a^3-b^3}{ab+3a^2}\leq \frac{6a^3-ab(a+b)}{ab+3a^2}=\frac{6a^2-ab-b^2}{3a+b}=\frac{(3a+b)(2a-b)}{3a+b}=2a-b$

Tương tự:

$\frac{5b^3-c^3}{bc+3b^2}\leq 2b-c; \frac{5c^3-a^3}{ca+3c^2}\leq 2c-a$

Cộng theo vế:

$\Rightarrow \text{VT}\leq a+b+c=3$

Ta có đpcm

Dấu "=" xảy ra khi $a=b=c=1$

24 tháng 12 2017

cái này dễ thôi, Áp dụng bđt cô si ta có 

\(\sqrt[3]{a+3b}\le\frac{a+3b+1+1}{3}\)

tương tự và + vào ta có \(A\le\frac{4\left(a+b+c\right)+6}{3}=3\) (đpcm)

dấu = xảy ra <=> a=b=c=1/4

15 tháng 2 2021

Bổ đề: \(a^3+b^3+c^3\ge\dfrac{1}{9}\left(a+b+c\right)^3\) \(\left(\forall a,b,c>0\right)\)

chứng minh bổ đề: \(\Sigma_{cyc}\left(\dfrac{a^3}{a^3+b^3+c^3}\right)+\dfrac{1}{3}+\dfrac{1}{3}\ge3\sqrt[3]{\left(\Pi_{cyc}\dfrac{a^3}{a^3+b^3+c^3}\right).\dfrac{1}{3}.\dfrac{1}{3}}\)

hoán vị theo a,b,c

ta được: \(3\ge\dfrac{3\left(a+b+c\right)}{\sqrt[3]{9.\left(a^3+b^3+c^3\right)}}\)

mũ 3 hai vế ta có được bất đẳng thức bổ đề: \(a^3+b^3+c^3\ge\dfrac{1}{9}\left(a+b+c\right)^3\)

Áp dụng bất C-S: 

\(\sqrt{a^3+3b}+\sqrt{b^3+3c}+\sqrt{c^3+3a}\ge\sqrt{\left(1+1+1\right)\left(a^3+b^3+c^3+3a+3b+3c\right)}\)

\(\ge\sqrt{3.\left[3+3\left(a+b+c\right)\right]}=\sqrt{36}=6\)

Dấu "=" xảy ra tại a=b=c=1

10 tháng 9 2017

Sang học 24 tìm ai tên Perfect Blue nhé t làm bên đó rồi đưa link thì lỗi ==" , tìm tên đăng nhập  springtime ấy

10 tháng 9 2017

Chào bác Thắng

AH
Akai Haruma
Giáo viên
27 tháng 8 2021

Lời giải:
Theo hệ quả quen thuộc của bđt AM-GM:
$(a+b+c)^2\leq 3(a^2+b^2+c^2)\leq 9$

$\Rightarrow a+b+c\leq 3$ (đpcm)

Từ đây ta có:

\(E\leq \frac{a}{\sqrt[3]{(a+b+c)a+bc}}+\frac{b}{\sqrt[3]{(a+b+c)b+ac}}+\frac{c}{\sqrt[3]{c(a+b+c)+ab}}\)

\(=\frac{a}{\sqrt[3]{(a+b)(a+c)}}+\frac{b}{\sqrt[3]{(b+c)(b+a)}}+\frac{c}{\sqrt[3]{(c+a)(c+b)}}\)

\(\leq \frac{\sqrt[3]{2}}{3}(\frac{a}{2}+\frac{a}{a+b}+\frac{a}{a+c})+\frac{\sqrt[3]{2}}{3}(\frac{b}{2}+\frac{b}{b+a}+\frac{b}{b+c})+\frac{\sqrt[3]{2}}{3}(\frac{c}{2}+\frac{c}{c+a}+\frac{c}{c+b})\)

\(=\frac{\sqrt[3]{2}(a+b+c)}{6}+\frac{\sqrt[3]{2}}{3}(\frac{a+b}{a+b}+\frac{b+c}{b+c}+\frac{c+a}{c+a})\leq \frac{3\sqrt[3]{2}}{2}\)

Vậy.................